Integer Pythagoras

You have a right triangle that has integer sides. Read the following statements about that triangle:

[ 1 ] [1] At least one of the sides of that triangle must be divisible by 3 3 .
[ 2 ] [2] At least one of the sides of that triangle must be divisible by 4 4 .
[ 3 ] [3] At least one of the sides of that triangle must be divisible by 5 5 .

Which of these statements is(are) correct?


Note : This problem is a part of this set .

Only [ 1 ] [1] [ 2 ] [2] and [ 3 ] [3] [ 1 ] [1] , [ 2 ] [2] , and [ 3 ] [3] None of them are correct

This section requires Javascript.
You are seeing this because something didn't load right. We suggest you, (a) try refreshing the page, (b) enabling javascript if it is disabled on your browser and, finally, (c) loading the non-javascript version of this page . We're sorry about the hassle.

6 solutions

Ivan Koswara
Apr 9, 2014

Suppose our triple is a , b , c a,b,c ; we know that a 2 + b 2 = c 2 a^2 + b^2 = c^2 .

Suppose none of a , b , c a,b,c is divisible by 3 3 . Notice that all squares are congruent to 0 , 1 ( m o d 3 ) 0,1 \pmod 3 ; moreover, if a number n n is not divisible by 3 3 , then n 2 n^2 is not divisible by 3 3 , so it must be congruent to 1 ( m o d 3 ) 1 \pmod 3 . But adding together, a 2 + b 2 1 + 1 2 ≢ 1 c ( m o d 3 ) a^2 + b^2 \equiv 1 + 1 \equiv 2 \not\equiv 1 \equiv c \pmod 3 , contradiction.

We can do a similar method for divisibility by 5 5 . We'll get 0 , 1 , 4 ( m o d 5 ) 0,1,4 \pmod 5 , and throwing away 0 0 , we have 1 , 4 ( m o d 5 ) 1,4 \pmod 5 . Thus by checking all four cases, a 2 + b 2 2 , 5 , 8 0 , 2 , 3 ( m o d 5 ) a^2 + b^2 \equiv 2, 5, 8 \equiv 0, 2, 3 \pmod 5 , which will never be congruent to c 2 c^2 .

For divisibility by 4 4 , this is a little tougher. We cannot just take quadratic residues modulo 4 4 , because 2 ≢ 0 ( m o d 4 ) 2 \not\equiv 0 \pmod 4 but 2 2 0 ( m o d 4 ) 2^2 \equiv 0 \pmod 4 . Instead, we need to take modulo 8 8 ; if n n is not divisible by 4 4 , then n 2 n^2 is not divisible by 8 8 . Also, the residues of squares modulo 8 8 are 0 , 1 , 4 0,1,4 ; throwing 0 0 , we have 1 , 4 ( m o d 8 ) 1,4 \pmod 8 . Thus we use the same method as with modulo 5 5 , getting c 2 2 , 5 , 8 0 , 2 , 5 ( m o d 8 ) c^2 \equiv 2,5,8 \equiv 0,2,5 \pmod 8 , which is also impossible.

Thus we have derived a contradiction from assuming that all side lengths are not divisible by any of 3 , 4 , 5 3,4,5 . Thus all three statements are correct .

Nice explanation

Mardokay Mosazghi - 7 years, 2 months ago

2nd para. should be 0,1,4 mod 5

Justin Leung - 2 years, 6 months ago

In reply to Ivan Koswara what about 7 the correct answer states all there are correct, please enlighten me about that

Somesh Patil - 7 years, 2 months ago

Log in to reply

Each statement refers to "at least one side", but not necessarily different sides. In this case, the 7 7 is not used in any of the statements, but the 24 24 is used twice instead. There's no problem with that. If the statement had been "one side is divisible by 3 3 , another side is divisible by 4 4 , and the last side is divisible by 5 5 " instead, then that statement is wrong.

Ivan Koswara - 7 years, 2 months ago

but what about 7, 24 25 triplet people it doesnt follows the same

Somesh Patil - 7 years, 2 months ago

Log in to reply

  • 24 24 is divisible by 3 3 .
  • 24 24 is divisible by 4 4 .
  • 25 25 is divisible by 5 5 .
Thus all three statements are correct.

Ivan Koswara - 7 years, 2 months ago
Eddie The Head
Apr 9, 2014

We know that Pythogorean triples can only be of the form k ( m 2 n 2 ) , 2 k m n k(m^{2} - n^{2}), 2kmn and k ( m 2 + n 2 ) k(m^{2}+n^{2}) .

Checking the divisibility with respect to different numbers given can lead us to the conclusion :

Case 1. \textbf{Case 1.} First we check the divisibility by 3 3 . If m n ( m o d 3 ) m \equiv n \pmod{3} ,then 3 3 divides m 2 n 2 m^{2} - n^{2} .

Also if any of m m or n n is divisible by 3 3 we are done since 3 2 k m n 3 | 2kmn .

So the only case left to prove is m 2 ( m o d 3 ) m\equiv 2 \pmod{3} and n 1 ( m o d 3 ) n\equiv 1 \pmod{3} (Without the loss of generality).

In this case clearly m 2 n 2 4 1 3 0 ( m o d 3 ) m^{2} - n^{2} \equiv 4-1 \equiv 3 \equiv 0 \pmod{3} .

hence the sides of the triangle are always divisible by 3.

The cases for 4 4 and 5 5 can also be nailed down using similar arguments.

This triangle is a right triangle.so First let us assign common Pythagorean Triples 3 , 4 , 5 3,4,5 And we know that the numbers 3 , 4 , 5 3,4,5 meet the criteria of been divisible by 3,4 and 5 thus the answer is 1 , 2.3 \boxed{1,2.3}

If I understand your solution correctly, this assumes that every Pythagorean triple is of the form ( 3 n , 4 n , 5 n ) (3n, 4n, 5n) , which is clearly false; many other families exist (such as the ( 8 , 15 , 17 ) (8, 15, 17) family). For a full solution, one must prove that every possible Pythagorean triple contains at least one number divisible by 3, etc.

Lee Wall - 7 years, 2 months ago

Log in to reply

Sorry I didn't realize that but thanks

Mardokay Mosazghi - 7 years, 2 months ago

15 is divisible by 3, 8 is divisible by 4, and 15 is divisible by 5.

Michael Mendrin - 7 years, 2 months ago

Log in to reply

I know, I was simply pointing out that because the ( 3 , 4 , 5 ) (3,4,5) triple satisfies the conditions does not imply that every triple satisfies the conditions.

Lee Wall - 7 years, 2 months ago

Log in to reply

@Lee Wall Oh, sorry, you're right. I misread.

Michael Mendrin - 7 years, 2 months ago

All Pythagorean triples can be expressed as k ( m 2 n 2 ) , 2 k m n , k ( m 2 + n 2 ) { k(m }^{ 2 }-{ n }^{ 2 }),\quad 2kmn,\quad k({ m }^{ 2 }+{ n }^{ 2 }) , where k, m, n are positive integers. Letting m, n = {0,1,2,3,4}, it can be seen that all 25 possible outcomes have numbers divisible by 3, 4, and 5.

Michael Mendrin - 7 years, 2 months ago

Log in to reply

How did u arrive at this conclusion? Can u please state a proof?

Vishal Ajwani - 7 years, 2 months ago

Log in to reply

Working out all 25 possible outcomes is a matter of arithmetic. However, the formula for Pythagorean triples is well-known. It can be derived, but that's a separate problem and proof. You can look up Wiki for such a proof, under "Pythagorean Triple"

Michael Mendrin - 7 years, 2 months ago

Log in to reply

@Michael Mendrin Thanks Sir

Vishal Ajwani - 7 years, 2 months ago

@Michael Mendrin Yeah that's a big proof the outcome of which is that pythogorean triplets are always of the form stated by @Michael Mendrin ...The theorem is also conversely true....

Eddie The Head - 7 years, 2 months ago
Jubayer Nirjhor
May 3, 2014

Just filter the equation x 2 + y 2 = z 2 x^2+y^2=z^2 with the quadratic residues modulo 3 , 4 , 5 3,4,5 and see that all of the statements are correct.

Cha Fi
Apr 15, 2014

we could have a triangle with 3²+4²=5² then one side is divisible by 3 the other by 4 and the last one by 5 so all the statements are correct

What about 5, 12 and 13?

Shubhrajit Sadhukhan - 7 months ago
Finn Hulse
Apr 8, 2014

I'm not sure how to prove it, but looking through the first 50 or so Pythagorean Triples, we can see that all of them have terms divisible by three, four, and five.

So I think you have made a conjecture, which means you noticed the relations between Pythagorean Triples but it cannot immediately prove that it is true for all Pythagorean Triples. Similarly, you can read Goldbach Conjecture , we can see that many numbers are true for Goldbach's Conjecture but it is still remain unsolved until this moment. What I want to say is, making a conjecture may be a good method and I sometimes do it in Mathematics Competition but make sure to read other brilliantics' solution on this problem to get a better and stronger prove on it.

Christopher Boo - 7 years, 2 months ago

Log in to reply

Yeah. Thanks!

Finn Hulse - 7 years, 2 months ago

0 pending reports

×

Problem Loading...

Note Loading...

Set Loading...